Why do the Borwein integrals stop being $frac{pi}{2}$?












6












$begingroup$


I just received the book "single digits - In praise of Small Numbers" by Marc Chamberland.



In this book, he gives an interesting integral



$$displaystyle int_0^infty dfrac{sin x}{x} = dfrac{pi}{2}$$



$$displaystyle int_0^infty dfrac{sin(x)}{x}dfrac{sin(x/3)}{x/3} = dfrac{pi}{2}$$



$$displaystyle int_0^infty dfrac{sin(x)}{x}dfrac{sin(x/3)}{x/3}dfrac{sin(x/5)}{x/5} = dfrac{pi}{2}$$



$$displaystyle int_0^infty dfrac{sin(x)}{x}dfrac{sin(x/3)}{x/3}dfrac{sin(x/5)}{x/5}dfrac{sin(x/7)}{x/7} = dfrac{pi}{2}$$



$$displaystyle int_0^infty dfrac{sin(x)}{x}dfrac{sin(x/3)}{x/3}dfrac{sin(x/5)}{x/5}dfrac{sin(x/7)}{x/7} dfrac{sin(x/9)}{x/9}= dfrac{pi}{2}$$



$$displaystyle int_0^infty dfrac{sin(x)}{x}dfrac{sin(x/3)}{x/3}dfrac{sin(x/5)}{x/5}dfrac{sin(x/7)}{x/7} dfrac{sin(x/9)}{x/9}dfrac{sin(x/11)}{x/11}= dfrac{pi}{2}$$



$$displaystyle int_0^infty dfrac{sin(x)}{x}dfrac{sin(x/3)}{x/3}dfrac{sin(x/5)}{x/5}dfrac{sin(x/7)}{x/7} dfrac{sin(x/9)}{x/9}dfrac{sin(x/11)}{x/11}dfrac{sin(x/13)}{x/13} = dfrac{pi}{2}$$



At this point, it is tempting to speculate that this pattern goes on forever, but we run into problems and this is another example of jumping to conclusions too soon.



$$displaystyle int_0^infty dfrac{sin(x)}{x}dfrac{sin(x/3)}{x/3}dfrac{sin(x/5)}{x/5}dfrac{sin(x/7)}{x/7} dfrac{sin(x/9)}{x/9}dfrac{sin(x/11)}{x/11}dfrac{sin(x/13)}{x/13}dfrac{sin(x/15)}{x/15} = dfrac{467807924713440738696537864469 pi }{935615849440640907310521750000}$$



I calculated the next several and they are nice approximations to the results above, but not that result




  • $$dfrac{17708695183056190642497315530628422295569865119 pi }{35417390788301195294898352987527510935040000000}$$

  • $$dfrac{8096799621940897567828686854312535486311061114550605367511653 pi }{16193600755941299921751838065715269433640150152124763150000000}$$

  • $$dfrac{2051563935160591194337436768610392837217226815379395891838337765936509 pi }{4103129007448718822870650414175026723860506854636748901313920000000000}$$


  • $$dfrac{37193167701690492344448194533283488902041049236760438302965167901187323851384840067287863 pi }{74386376780038719358535506076609218130495936637120586884474907521986965251324791250000000}$$



He states "The explanation for this change is a bit technical, but the critical reason is that $dfrac{1}{3} + dfrac{1}{5} + ldots + dfrac{1}{13} lt 1$, whereas, adding the next term $frac{1}{15}$ pushes the sum over $1$, making a difference in the value of the integral."



He does not mention the researcher, but I'd like to know what is a "bit technical" explanation or if there is a more analytical or mathematical rationale or a reference to the research?










share|cite|improve this question











$endgroup$








  • 5




    $begingroup$
    This is an interesting example of "jumping to a conclusion." (+1)
    $endgroup$
    – Mark Viola
    Dec 4 '18 at 16:48






  • 1




    $begingroup$
    A derivation of the integral is given on Wiki::Borwein integral which explains the result (i.e. why it suddenly fails to hold true once the sum of some series gets large enough)
    $endgroup$
    – Winther
    Dec 4 '18 at 17:02








  • 1




    $begingroup$
    A paper in the same spirit (formulas that holds for the first $N$ integers and then suddenly fails) might also be of interest "Fun with large numbers" by R. Baillie.
    $endgroup$
    – Winther
    Dec 4 '18 at 17:05






  • 2




    $begingroup$
    @Winther Hi Hans. Happy Holidays. Thank you for the comments and embedded references! Much appreciated. -Mark
    $endgroup$
    – Mark Viola
    Dec 4 '18 at 19:03






  • 1




    $begingroup$
    There's a variant of this identity that holds until 15,341,178,777,673,149,429,167,740,440,969,249,338,310,888 but fails at the next numbers :) see here.
    $endgroup$
    – Arnaud D.
    Dec 4 '18 at 19:39
















6












$begingroup$


I just received the book "single digits - In praise of Small Numbers" by Marc Chamberland.



In this book, he gives an interesting integral



$$displaystyle int_0^infty dfrac{sin x}{x} = dfrac{pi}{2}$$



$$displaystyle int_0^infty dfrac{sin(x)}{x}dfrac{sin(x/3)}{x/3} = dfrac{pi}{2}$$



$$displaystyle int_0^infty dfrac{sin(x)}{x}dfrac{sin(x/3)}{x/3}dfrac{sin(x/5)}{x/5} = dfrac{pi}{2}$$



$$displaystyle int_0^infty dfrac{sin(x)}{x}dfrac{sin(x/3)}{x/3}dfrac{sin(x/5)}{x/5}dfrac{sin(x/7)}{x/7} = dfrac{pi}{2}$$



$$displaystyle int_0^infty dfrac{sin(x)}{x}dfrac{sin(x/3)}{x/3}dfrac{sin(x/5)}{x/5}dfrac{sin(x/7)}{x/7} dfrac{sin(x/9)}{x/9}= dfrac{pi}{2}$$



$$displaystyle int_0^infty dfrac{sin(x)}{x}dfrac{sin(x/3)}{x/3}dfrac{sin(x/5)}{x/5}dfrac{sin(x/7)}{x/7} dfrac{sin(x/9)}{x/9}dfrac{sin(x/11)}{x/11}= dfrac{pi}{2}$$



$$displaystyle int_0^infty dfrac{sin(x)}{x}dfrac{sin(x/3)}{x/3}dfrac{sin(x/5)}{x/5}dfrac{sin(x/7)}{x/7} dfrac{sin(x/9)}{x/9}dfrac{sin(x/11)}{x/11}dfrac{sin(x/13)}{x/13} = dfrac{pi}{2}$$



At this point, it is tempting to speculate that this pattern goes on forever, but we run into problems and this is another example of jumping to conclusions too soon.



$$displaystyle int_0^infty dfrac{sin(x)}{x}dfrac{sin(x/3)}{x/3}dfrac{sin(x/5)}{x/5}dfrac{sin(x/7)}{x/7} dfrac{sin(x/9)}{x/9}dfrac{sin(x/11)}{x/11}dfrac{sin(x/13)}{x/13}dfrac{sin(x/15)}{x/15} = dfrac{467807924713440738696537864469 pi }{935615849440640907310521750000}$$



I calculated the next several and they are nice approximations to the results above, but not that result




  • $$dfrac{17708695183056190642497315530628422295569865119 pi }{35417390788301195294898352987527510935040000000}$$

  • $$dfrac{8096799621940897567828686854312535486311061114550605367511653 pi }{16193600755941299921751838065715269433640150152124763150000000}$$

  • $$dfrac{2051563935160591194337436768610392837217226815379395891838337765936509 pi }{4103129007448718822870650414175026723860506854636748901313920000000000}$$


  • $$dfrac{37193167701690492344448194533283488902041049236760438302965167901187323851384840067287863 pi }{74386376780038719358535506076609218130495936637120586884474907521986965251324791250000000}$$



He states "The explanation for this change is a bit technical, but the critical reason is that $dfrac{1}{3} + dfrac{1}{5} + ldots + dfrac{1}{13} lt 1$, whereas, adding the next term $frac{1}{15}$ pushes the sum over $1$, making a difference in the value of the integral."



He does not mention the researcher, but I'd like to know what is a "bit technical" explanation or if there is a more analytical or mathematical rationale or a reference to the research?










share|cite|improve this question











$endgroup$








  • 5




    $begingroup$
    This is an interesting example of "jumping to a conclusion." (+1)
    $endgroup$
    – Mark Viola
    Dec 4 '18 at 16:48






  • 1




    $begingroup$
    A derivation of the integral is given on Wiki::Borwein integral which explains the result (i.e. why it suddenly fails to hold true once the sum of some series gets large enough)
    $endgroup$
    – Winther
    Dec 4 '18 at 17:02








  • 1




    $begingroup$
    A paper in the same spirit (formulas that holds for the first $N$ integers and then suddenly fails) might also be of interest "Fun with large numbers" by R. Baillie.
    $endgroup$
    – Winther
    Dec 4 '18 at 17:05






  • 2




    $begingroup$
    @Winther Hi Hans. Happy Holidays. Thank you for the comments and embedded references! Much appreciated. -Mark
    $endgroup$
    – Mark Viola
    Dec 4 '18 at 19:03






  • 1




    $begingroup$
    There's a variant of this identity that holds until 15,341,178,777,673,149,429,167,740,440,969,249,338,310,888 but fails at the next numbers :) see here.
    $endgroup$
    – Arnaud D.
    Dec 4 '18 at 19:39














6












6








6


2



$begingroup$


I just received the book "single digits - In praise of Small Numbers" by Marc Chamberland.



In this book, he gives an interesting integral



$$displaystyle int_0^infty dfrac{sin x}{x} = dfrac{pi}{2}$$



$$displaystyle int_0^infty dfrac{sin(x)}{x}dfrac{sin(x/3)}{x/3} = dfrac{pi}{2}$$



$$displaystyle int_0^infty dfrac{sin(x)}{x}dfrac{sin(x/3)}{x/3}dfrac{sin(x/5)}{x/5} = dfrac{pi}{2}$$



$$displaystyle int_0^infty dfrac{sin(x)}{x}dfrac{sin(x/3)}{x/3}dfrac{sin(x/5)}{x/5}dfrac{sin(x/7)}{x/7} = dfrac{pi}{2}$$



$$displaystyle int_0^infty dfrac{sin(x)}{x}dfrac{sin(x/3)}{x/3}dfrac{sin(x/5)}{x/5}dfrac{sin(x/7)}{x/7} dfrac{sin(x/9)}{x/9}= dfrac{pi}{2}$$



$$displaystyle int_0^infty dfrac{sin(x)}{x}dfrac{sin(x/3)}{x/3}dfrac{sin(x/5)}{x/5}dfrac{sin(x/7)}{x/7} dfrac{sin(x/9)}{x/9}dfrac{sin(x/11)}{x/11}= dfrac{pi}{2}$$



$$displaystyle int_0^infty dfrac{sin(x)}{x}dfrac{sin(x/3)}{x/3}dfrac{sin(x/5)}{x/5}dfrac{sin(x/7)}{x/7} dfrac{sin(x/9)}{x/9}dfrac{sin(x/11)}{x/11}dfrac{sin(x/13)}{x/13} = dfrac{pi}{2}$$



At this point, it is tempting to speculate that this pattern goes on forever, but we run into problems and this is another example of jumping to conclusions too soon.



$$displaystyle int_0^infty dfrac{sin(x)}{x}dfrac{sin(x/3)}{x/3}dfrac{sin(x/5)}{x/5}dfrac{sin(x/7)}{x/7} dfrac{sin(x/9)}{x/9}dfrac{sin(x/11)}{x/11}dfrac{sin(x/13)}{x/13}dfrac{sin(x/15)}{x/15} = dfrac{467807924713440738696537864469 pi }{935615849440640907310521750000}$$



I calculated the next several and they are nice approximations to the results above, but not that result




  • $$dfrac{17708695183056190642497315530628422295569865119 pi }{35417390788301195294898352987527510935040000000}$$

  • $$dfrac{8096799621940897567828686854312535486311061114550605367511653 pi }{16193600755941299921751838065715269433640150152124763150000000}$$

  • $$dfrac{2051563935160591194337436768610392837217226815379395891838337765936509 pi }{4103129007448718822870650414175026723860506854636748901313920000000000}$$


  • $$dfrac{37193167701690492344448194533283488902041049236760438302965167901187323851384840067287863 pi }{74386376780038719358535506076609218130495936637120586884474907521986965251324791250000000}$$



He states "The explanation for this change is a bit technical, but the critical reason is that $dfrac{1}{3} + dfrac{1}{5} + ldots + dfrac{1}{13} lt 1$, whereas, adding the next term $frac{1}{15}$ pushes the sum over $1$, making a difference in the value of the integral."



He does not mention the researcher, but I'd like to know what is a "bit technical" explanation or if there is a more analytical or mathematical rationale or a reference to the research?










share|cite|improve this question











$endgroup$




I just received the book "single digits - In praise of Small Numbers" by Marc Chamberland.



In this book, he gives an interesting integral



$$displaystyle int_0^infty dfrac{sin x}{x} = dfrac{pi}{2}$$



$$displaystyle int_0^infty dfrac{sin(x)}{x}dfrac{sin(x/3)}{x/3} = dfrac{pi}{2}$$



$$displaystyle int_0^infty dfrac{sin(x)}{x}dfrac{sin(x/3)}{x/3}dfrac{sin(x/5)}{x/5} = dfrac{pi}{2}$$



$$displaystyle int_0^infty dfrac{sin(x)}{x}dfrac{sin(x/3)}{x/3}dfrac{sin(x/5)}{x/5}dfrac{sin(x/7)}{x/7} = dfrac{pi}{2}$$



$$displaystyle int_0^infty dfrac{sin(x)}{x}dfrac{sin(x/3)}{x/3}dfrac{sin(x/5)}{x/5}dfrac{sin(x/7)}{x/7} dfrac{sin(x/9)}{x/9}= dfrac{pi}{2}$$



$$displaystyle int_0^infty dfrac{sin(x)}{x}dfrac{sin(x/3)}{x/3}dfrac{sin(x/5)}{x/5}dfrac{sin(x/7)}{x/7} dfrac{sin(x/9)}{x/9}dfrac{sin(x/11)}{x/11}= dfrac{pi}{2}$$



$$displaystyle int_0^infty dfrac{sin(x)}{x}dfrac{sin(x/3)}{x/3}dfrac{sin(x/5)}{x/5}dfrac{sin(x/7)}{x/7} dfrac{sin(x/9)}{x/9}dfrac{sin(x/11)}{x/11}dfrac{sin(x/13)}{x/13} = dfrac{pi}{2}$$



At this point, it is tempting to speculate that this pattern goes on forever, but we run into problems and this is another example of jumping to conclusions too soon.



$$displaystyle int_0^infty dfrac{sin(x)}{x}dfrac{sin(x/3)}{x/3}dfrac{sin(x/5)}{x/5}dfrac{sin(x/7)}{x/7} dfrac{sin(x/9)}{x/9}dfrac{sin(x/11)}{x/11}dfrac{sin(x/13)}{x/13}dfrac{sin(x/15)}{x/15} = dfrac{467807924713440738696537864469 pi }{935615849440640907310521750000}$$



I calculated the next several and they are nice approximations to the results above, but not that result




  • $$dfrac{17708695183056190642497315530628422295569865119 pi }{35417390788301195294898352987527510935040000000}$$

  • $$dfrac{8096799621940897567828686854312535486311061114550605367511653 pi }{16193600755941299921751838065715269433640150152124763150000000}$$

  • $$dfrac{2051563935160591194337436768610392837217226815379395891838337765936509 pi }{4103129007448718822870650414175026723860506854636748901313920000000000}$$


  • $$dfrac{37193167701690492344448194533283488902041049236760438302965167901187323851384840067287863 pi }{74386376780038719358535506076609218130495936637120586884474907521986965251324791250000000}$$



He states "The explanation for this change is a bit technical, but the critical reason is that $dfrac{1}{3} + dfrac{1}{5} + ldots + dfrac{1}{13} lt 1$, whereas, adding the next term $frac{1}{15}$ pushes the sum over $1$, making a difference in the value of the integral."



He does not mention the researcher, but I'd like to know what is a "bit technical" explanation or if there is a more analytical or mathematical rationale or a reference to the research?







reference-request definite-integrals divergent-series






share|cite|improve this question















share|cite|improve this question













share|cite|improve this question




share|cite|improve this question








edited Dec 4 '18 at 16:43









Xander Henderson

14.2k103554




14.2k103554










asked Dec 4 '18 at 16:22









MooMoo

5,53131020




5,53131020








  • 5




    $begingroup$
    This is an interesting example of "jumping to a conclusion." (+1)
    $endgroup$
    – Mark Viola
    Dec 4 '18 at 16:48






  • 1




    $begingroup$
    A derivation of the integral is given on Wiki::Borwein integral which explains the result (i.e. why it suddenly fails to hold true once the sum of some series gets large enough)
    $endgroup$
    – Winther
    Dec 4 '18 at 17:02








  • 1




    $begingroup$
    A paper in the same spirit (formulas that holds for the first $N$ integers and then suddenly fails) might also be of interest "Fun with large numbers" by R. Baillie.
    $endgroup$
    – Winther
    Dec 4 '18 at 17:05






  • 2




    $begingroup$
    @Winther Hi Hans. Happy Holidays. Thank you for the comments and embedded references! Much appreciated. -Mark
    $endgroup$
    – Mark Viola
    Dec 4 '18 at 19:03






  • 1




    $begingroup$
    There's a variant of this identity that holds until 15,341,178,777,673,149,429,167,740,440,969,249,338,310,888 but fails at the next numbers :) see here.
    $endgroup$
    – Arnaud D.
    Dec 4 '18 at 19:39














  • 5




    $begingroup$
    This is an interesting example of "jumping to a conclusion." (+1)
    $endgroup$
    – Mark Viola
    Dec 4 '18 at 16:48






  • 1




    $begingroup$
    A derivation of the integral is given on Wiki::Borwein integral which explains the result (i.e. why it suddenly fails to hold true once the sum of some series gets large enough)
    $endgroup$
    – Winther
    Dec 4 '18 at 17:02








  • 1




    $begingroup$
    A paper in the same spirit (formulas that holds for the first $N$ integers and then suddenly fails) might also be of interest "Fun with large numbers" by R. Baillie.
    $endgroup$
    – Winther
    Dec 4 '18 at 17:05






  • 2




    $begingroup$
    @Winther Hi Hans. Happy Holidays. Thank you for the comments and embedded references! Much appreciated. -Mark
    $endgroup$
    – Mark Viola
    Dec 4 '18 at 19:03






  • 1




    $begingroup$
    There's a variant of this identity that holds until 15,341,178,777,673,149,429,167,740,440,969,249,338,310,888 but fails at the next numbers :) see here.
    $endgroup$
    – Arnaud D.
    Dec 4 '18 at 19:39








5




5




$begingroup$
This is an interesting example of "jumping to a conclusion." (+1)
$endgroup$
– Mark Viola
Dec 4 '18 at 16:48




$begingroup$
This is an interesting example of "jumping to a conclusion." (+1)
$endgroup$
– Mark Viola
Dec 4 '18 at 16:48




1




1




$begingroup$
A derivation of the integral is given on Wiki::Borwein integral which explains the result (i.e. why it suddenly fails to hold true once the sum of some series gets large enough)
$endgroup$
– Winther
Dec 4 '18 at 17:02






$begingroup$
A derivation of the integral is given on Wiki::Borwein integral which explains the result (i.e. why it suddenly fails to hold true once the sum of some series gets large enough)
$endgroup$
– Winther
Dec 4 '18 at 17:02






1




1




$begingroup$
A paper in the same spirit (formulas that holds for the first $N$ integers and then suddenly fails) might also be of interest "Fun with large numbers" by R. Baillie.
$endgroup$
– Winther
Dec 4 '18 at 17:05




$begingroup$
A paper in the same spirit (formulas that holds for the first $N$ integers and then suddenly fails) might also be of interest "Fun with large numbers" by R. Baillie.
$endgroup$
– Winther
Dec 4 '18 at 17:05




2




2




$begingroup$
@Winther Hi Hans. Happy Holidays. Thank you for the comments and embedded references! Much appreciated. -Mark
$endgroup$
– Mark Viola
Dec 4 '18 at 19:03




$begingroup$
@Winther Hi Hans. Happy Holidays. Thank you for the comments and embedded references! Much appreciated. -Mark
$endgroup$
– Mark Viola
Dec 4 '18 at 19:03




1




1




$begingroup$
There's a variant of this identity that holds until 15,341,178,777,673,149,429,167,740,440,969,249,338,310,888 but fails at the next numbers :) see here.
$endgroup$
– Arnaud D.
Dec 4 '18 at 19:39




$begingroup$
There's a variant of this identity that holds until 15,341,178,777,673,149,429,167,740,440,969,249,338,310,888 but fails at the next numbers :) see here.
$endgroup$
– Arnaud D.
Dec 4 '18 at 19:39










0






active

oldest

votes











Your Answer





StackExchange.ifUsing("editor", function () {
return StackExchange.using("mathjaxEditing", function () {
StackExchange.MarkdownEditor.creationCallbacks.add(function (editor, postfix) {
StackExchange.mathjaxEditing.prepareWmdForMathJax(editor, postfix, [["$", "$"], ["\\(","\\)"]]);
});
});
}, "mathjax-editing");

StackExchange.ready(function() {
var channelOptions = {
tags: "".split(" "),
id: "69"
};
initTagRenderer("".split(" "), "".split(" "), channelOptions);

StackExchange.using("externalEditor", function() {
// Have to fire editor after snippets, if snippets enabled
if (StackExchange.settings.snippets.snippetsEnabled) {
StackExchange.using("snippets", function() {
createEditor();
});
}
else {
createEditor();
}
});

function createEditor() {
StackExchange.prepareEditor({
heartbeatType: 'answer',
autoActivateHeartbeat: false,
convertImagesToLinks: true,
noModals: true,
showLowRepImageUploadWarning: true,
reputationToPostImages: 10,
bindNavPrevention: true,
postfix: "",
imageUploader: {
brandingHtml: "Powered by u003ca class="icon-imgur-white" href="https://imgur.com/"u003eu003c/au003e",
contentPolicyHtml: "User contributions licensed under u003ca href="https://creativecommons.org/licenses/by-sa/3.0/"u003ecc by-sa 3.0 with attribution requiredu003c/au003e u003ca href="https://stackoverflow.com/legal/content-policy"u003e(content policy)u003c/au003e",
allowUrls: true
},
noCode: true, onDemand: true,
discardSelector: ".discard-answer"
,immediatelyShowMarkdownHelp:true
});


}
});














draft saved

draft discarded


















StackExchange.ready(
function () {
StackExchange.openid.initPostLogin('.new-post-login', 'https%3a%2f%2fmath.stackexchange.com%2fquestions%2f3025772%2fwhy-do-the-borwein-integrals-stop-being-frac-pi2%23new-answer', 'question_page');
}
);

Post as a guest















Required, but never shown

























0






active

oldest

votes








0






active

oldest

votes









active

oldest

votes






active

oldest

votes
















draft saved

draft discarded




















































Thanks for contributing an answer to Mathematics Stack Exchange!


  • Please be sure to answer the question. Provide details and share your research!

But avoid



  • Asking for help, clarification, or responding to other answers.

  • Making statements based on opinion; back them up with references or personal experience.


Use MathJax to format equations. MathJax reference.


To learn more, see our tips on writing great answers.




draft saved


draft discarded














StackExchange.ready(
function () {
StackExchange.openid.initPostLogin('.new-post-login', 'https%3a%2f%2fmath.stackexchange.com%2fquestions%2f3025772%2fwhy-do-the-borwein-integrals-stop-being-frac-pi2%23new-answer', 'question_page');
}
);

Post as a guest















Required, but never shown





















































Required, but never shown














Required, but never shown












Required, but never shown







Required, but never shown

































Required, but never shown














Required, but never shown












Required, but never shown







Required, but never shown







Popular posts from this blog

To store a contact into the json file from server.js file using a class in NodeJS

Redirect URL with Chrome Remote Debugging Android Devices

Dieringhausen